As explained in the video explanation for this question, the game set up provides two options:
#1 J: R __ G F: G/Y __ R/Y
#2: J: __ __ R/Y F: R/Y __ G
The question stem asks "if one set of the line 2 tickets is green . . ."
Well, in setup #2, none of the line 2 tickets can be green. So we know we are in setup #1.
Ok. So let's look at setup #1 more closely:
#1 J: R __ G F: G/Y __ R/Y
Answer choice (A) is immediately clear--the line 1 tickets for January ARE red.
Answer choice (B) cannot be true, because the line 3 tickets for January in setup #1 are green.
Answer choice (C) cannot be true, because the line 1 tickets for February in setup #1 are either green or yellow.
Answer choice (D) cannot be true, because the line 3 tickets for February in setup #1 are either red or yellow.
Answer choice (E) could be true--it is possible that the line 3 tickets for February in setup #1 are yellow, but it is also possible that these tickets are red. The question asks you what MUST be true. Answer choice (A) is correct.
Hope this helps! Please let us know if you have any additional questions.